6.7: L'Hopital's Rule (2024)

  1. Last updated
  2. Save as PDF
  • Page ID
    4659
  • \( \newcommand{\vecs}[1]{\overset { \scriptstyle \rightharpoonup} {\mathbf{#1}}}\)

    \( \newcommand{\vecd}[1]{\overset{-\!-\!\rightharpoonup}{\vphantom{a}\smash{#1}}} \)

    \( \newcommand{\id}{\mathrm{id}}\) \( \newcommand{\Span}{\mathrm{span}}\)

    ( \newcommand{\kernel}{\mathrm{null}\,}\) \( \newcommand{\range}{\mathrm{range}\,}\)

    \( \newcommand{\RealPart}{\mathrm{Re}}\) \( \newcommand{\ImaginaryPart}{\mathrm{Im}}\)

    \( \newcommand{\Argument}{\mathrm{Arg}}\) \( \newcommand{\norm}[1]{\| #1 \|}\)

    \( \newcommand{\inner}[2]{\langle #1, #2 \rangle}\)

    \( \newcommand{\Span}{\mathrm{span}}\)

    \( \newcommand{\id}{\mathrm{id}}\)

    \( \newcommand{\Span}{\mathrm{span}}\)

    \( \newcommand{\kernel}{\mathrm{null}\,}\)

    \( \newcommand{\range}{\mathrm{range}\,}\)

    \( \newcommand{\RealPart}{\mathrm{Re}}\)

    \( \newcommand{\ImaginaryPart}{\mathrm{Im}}\)

    \( \newcommand{\Argument}{\mathrm{Arg}}\)

    \( \newcommand{\norm}[1]{\| #1 \|}\)

    \( \newcommand{\inner}[2]{\langle #1, #2 \rangle}\)

    \( \newcommand{\Span}{\mathrm{span}}\) \( \newcommand{\AA}{\unicode[.8,0]{x212B}}\)

    \( \newcommand{\vectorA}[1]{\vec{#1}} % arrow\)

    \( \newcommand{\vectorAt}[1]{\vec{\text{#1}}} % arrow\)

    \( \newcommand{\vectorB}[1]{\overset { \scriptstyle \rightharpoonup} {\mathbf{#1}}}\)

    \( \newcommand{\vectorC}[1]{\textbf{#1}}\)

    \( \newcommand{\vectorD}[1]{\overrightarrow{#1}}\)

    \( \newcommand{\vectorDt}[1]{\overrightarrow{\text{#1}}}\)

    \( \newcommand{\vectE}[1]{\overset{-\!-\!\rightharpoonup}{\vphantom{a}\smash{\mathbf {#1}}}} \)

    \( \newcommand{\vecs}[1]{\overset { \scriptstyle \rightharpoonup} {\mathbf{#1}}}\)

    \( \newcommand{\vecd}[1]{\overset{-\!-\!\rightharpoonup}{\vphantom{a}\smash{#1}}} \)

    \(\newcommand{\avec}{\mathbf a}\) \(\newcommand{\bvec}{\mathbf b}\) \(\newcommand{\cvec}{\mathbf c}\) \(\newcommand{\dvec}{\mathbf d}\) \(\newcommand{\dtil}{\widetilde{\mathbf d}}\) \(\newcommand{\evec}{\mathbf e}\) \(\newcommand{\fvec}{\mathbf f}\) \(\newcommand{\nvec}{\mathbf n}\) \(\newcommand{\pvec}{\mathbf p}\) \(\newcommand{\qvec}{\mathbf q}\) \(\newcommand{\svec}{\mathbf s}\) \(\newcommand{\tvec}{\mathbf t}\) \(\newcommand{\uvec}{\mathbf u}\) \(\newcommand{\vvec}{\mathbf v}\) \(\newcommand{\wvec}{\mathbf w}\) \(\newcommand{\xvec}{\mathbf x}\) \(\newcommand{\yvec}{\mathbf y}\) \(\newcommand{\zvec}{\mathbf z}\) \(\newcommand{\rvec}{\mathbf r}\) \(\newcommand{\mvec}{\mathbf m}\) \(\newcommand{\zerovec}{\mathbf 0}\) \(\newcommand{\onevec}{\mathbf 1}\) \(\newcommand{\real}{\mathbb R}\) \(\newcommand{\twovec}[2]{\left[\begin{array}{r}#1 \\ #2 \end{array}\right]}\) \(\newcommand{\ctwovec}[2]{\left[\begin{array}{c}#1 \\ #2 \end{array}\right]}\) \(\newcommand{\threevec}[3]{\left[\begin{array}{r}#1 \\ #2 \\ #3 \end{array}\right]}\) \(\newcommand{\cthreevec}[3]{\left[\begin{array}{c}#1 \\ #2 \\ #3 \end{array}\right]}\) \(\newcommand{\fourvec}[4]{\left[\begin{array}{r}#1 \\ #2 \\ #3 \\ #4 \end{array}\right]}\) \(\newcommand{\cfourvec}[4]{\left[\begin{array}{c}#1 \\ #2 \\ #3 \\ #4 \end{array}\right]}\) \(\newcommand{\fivevec}[5]{\left[\begin{array}{r}#1 \\ #2 \\ #3 \\ #4 \\ #5 \\ \end{array}\right]}\) \(\newcommand{\cfivevec}[5]{\left[\begin{array}{c}#1 \\ #2 \\ #3 \\ #4 \\ #5 \\ \end{array}\right]}\) \(\newcommand{\mattwo}[4]{\left[\begin{array}{rr}#1 \amp #2 \\ #3 \amp #4 \\ \end{array}\right]}\) \(\newcommand{\laspan}[1]{\text{Span}\{#1\}}\) \(\newcommand{\bcal}{\cal B}\) \(\newcommand{\ccal}{\cal C}\) \(\newcommand{\scal}{\cal S}\) \(\newcommand{\wcal}{\cal W}\) \(\newcommand{\ecal}{\cal E}\) \(\newcommand{\coords}[2]{\left\{#1\right\}_{#2}}\) \(\newcommand{\gray}[1]{\color{gray}{#1}}\) \(\newcommand{\lgray}[1]{\color{lightgray}{#1}}\) \(\newcommand{\rank}{\operatorname{rank}}\) \(\newcommand{\row}{\text{Row}}\) \(\newcommand{\col}{\text{Col}}\) \(\renewcommand{\row}{\text{Row}}\) \(\newcommand{\nul}{\text{Nul}}\) \(\newcommand{\var}{\text{Var}}\) \(\newcommand{\corr}{\text{corr}}\) \(\newcommand{\len}[1]{\left|#1\right|}\) \(\newcommand{\bbar}{\overline{\bvec}}\) \(\newcommand{\bhat}{\widehat{\bvec}}\) \(\newcommand{\bperp}{\bvec^\perp}\) \(\newcommand{\xhat}{\widehat{\xvec}}\) \(\newcommand{\vhat}{\widehat{\vvec}}\) \(\newcommand{\uhat}{\widehat{\uvec}}\) \(\newcommand{\what}{\widehat{\wvec}}\) \(\newcommand{\Sighat}{\widehat{\Sigma}}\) \(\newcommand{\lt}{<}\) \(\newcommand{\gt}{>}\) \(\newcommand{\amp}{&}\) \(\definecolor{fillinmathshade}{gray}{0.9}\)

    While this chapter is devoted to learning techniques of integration, this section is not about integration. Rather, it is concerned with a technique of evaluating certain limits that will be useful in the following section, where integration is once more discussed.

    Our treatment of limits exposed us to "0/0", an indeterminate form. If \( \lim_{x\to c}f(x)=0\) and \( \lim_{x\to c} g(x) =0\), we do not conclude that \( \lim_{x\to c} f(x)/g(x)\) is \(0/0\); rather, we use \(0/0\) as notation to describe the fact that both the numerator and denominator approach 0. The expression 0/0 has no numeric value; other work must be done to evaluate the limit.

    Other indeterminate forms exist; they are: \(\infty/\infty\), \(0\cdot\infty\), \(\infty-\infty\), \(0^0\), \(1^\infty\) and \(\infty^0\). Just as "0/0" does not mean "divide 0 by 0," the expression "\(\infty/\infty\)" does not mean "divide infinity by infinity." Instead, it means "a quantity is growing without bound and is being divided by another quantity that is growing without bound." We cannot determine from such a statement what value, if any, results in the limit. Likewise, "\(0\cdot \infty\)" does not mean "multiply zero by infinity." Instead, it means "one quantity is shrinking to zero, and is being multiplied by a quantity that is growing without bound." We cannot determine from such a description what the result of such a limit will be.

    This section introduces L'Hôpital's Rule, a method of resolving limits that produce the indeterminate forms 0/0 and \(\infty/\infty\). We'll also show how algebraic manipulation can be used to convert other indeterminate expressions into one of these two forms so that our new rule can be applied.

    Theorem \(\PageIndex{1}\): L'Hôpital's Rule

    Let \(\lim_{x\to c}f(x) = 0\) and \(\lim_{x\to c}g(x)=0\), where \(f\) and \(g\) are differentiable functions on an open interval \(I\) containing \(c\), and \(g'(x)\neq 0\) on \(I\) except possibly at \(c\). Then

    $$ \lim_{x\to c} \frac{f(x)}{g(x)} = \lim_{x\to c} \frac{f'(x)}{g'(x)}.\]

    We demonstrate the use of L'Hôpital's Rule in the following examples; we will often use "LHR" as an abbreviation of "L'Hôpital's"

    Example \(\PageIndex{1}\): Using L'Hôpital's Rule

    Evaluate the following limits, using L'Hôpital's Rule as needed.

    1. \( \lim_{x\to0}\frac{\sin x}x\)
    2. \( \lim_{x\to 1}\frac{\sqrt{x+3}-2}{1-x}\)
    3. \( \lim_{x\to0}\frac{x^2}{1-\cos x}\)
    4. \( \lim_{x\to 2}\frac{x^2+x-6}{x^2-3x+2}\)

    Solution

    1. We proved this limit is 1 in Example \ref{ex_limit_sinx_prove} using the Squeeze Theorem. Here we use L'Hôpital's Rule to show its power. $$\lim_{x\to0}\frac{\sin x}x \stackrel{ \text{ by LHR } \ }{=} \lim_{x\to0} \frac{\cos x}{1}=1.$$
    2. \( \lim_{x\to 1}\frac{\sqrt{x+3}-2}{1-x} \stackrel{\ \text{ by LHR } \ }{=} \lim_{x \to 1} \frac{\frac12(x+3)^{-1/2}}{-1} =-\frac 14.\)
    3. \( \lim_{x\to 0}\frac{x^2}{1-\cos x} \stackrel{ \text{ by LHR } }{=} \lim_{x\to 0} \frac{2x}{\sin x}.\)
      This latter limit also evaluates to the 0/0 indeterminate form. To evaluate it, we apply L'Hôpital's Rule again.$$ \lim_{x\to 0} \frac{2x}{\sin x} \stackrel{ \text{ by LHR } }{=} \frac{2}{\cos x} = 2 .$$ Thus \( \lim_{x\to0}\frac{x^2}{1-\cos x}=2.\)
    4. We already know how to evaluate this limit; first factor the numerator and denominator. We then have: $$\lim_{x\to 2}\frac{x^2+x-6}{x^2-3x+2} = \lim_{x\to 2}\frac{(x-2)(x+3)}{(x-2)(x-1)} = \lim_{x\to 2}\frac{x+3}{x-1} = 5.$$ We now show how to solve this using L'Hôpital's Rule.$$\lim_{x\to 2}\frac{x^2+x-6}{x^2-3x+2}\stackrel{ \text{ by LHR } }{=} \lim_{x\to 2}\frac{2x+1}{2x-3} = 5.$$

    Note that at each step where L'Hôpital's Rule was applied, it was needed: the initial limit returned the indeterminate form of "\(0/0\)." If the initial limit returns, for example, 1/2, then L'Hôpital's Rule does not apply.

    The following theorem extends our initial version of L'Hôpital's Rule in two ways. It allows the technique to be applied to the indeterminate form \(\infty/\infty\) and to limits where \(x\) approaches \(\pm\infty\).

    Theorem \(\PageIndex{2}\): L'Hôpital's Rule, Part 2

    1. Let \(\lim_{x\to a}f(x) = \pm\infty\) and \(\lim_{x\to a}g(x)=\pm \infty\), where \(f\) and \(g\) are differentiable on an open interval \(I\) containing \(a\). Then $$\lim_{x\to a} \frac{f(x)}{g(x)} = \lim_{x\to a}\frac{f'(x)}{g'(x)}.$$
    2. Let \(f\) and \(g\) be differentiable functions on the open interval \((a,\infty)\) for some value \(a\), where \(g'(x)\neq 0\) on \((a,\infty)\) and \(\lim_{x\to\infty} f(x)/g(x)\) returns either 0/0 or \(\infty/\infty\). Then $$\lim_{x\to \infty} \frac{f(x)}{g(x)} = \lim_{x\to \infty}\frac{f'(x)}{g'(x)}.$$ A similar statement can be made for limits where \(x\) approaches \(-\infty\).

    Example \(\PageIndex{2}\): L'Hôpital's Rule with limits involving \(\infty\)

    Evaluate the following limits.

    $$ 1.\ \lim_{x\to\infty} \frac{3x^2-100x+2}{4x^2+5x-1000} \qquad\qquad 2. \ \lim_{x\to \infty}\frac{e^x}{x^3}.\]

    Solution

    1. We can evaluate this limit already using Theorem \ref{thm:lim_rational_fn_at_infty}; the answer is 3/4. We apply L'Hôpital's Rule to demonstrate its applicability. $$\lim_{x\to\infty} \frac{3x^2-100x+2}{4x^2+5x-1000}\stackrel{\ \text{ by LHR } \ }{=} \lim_{x\to\infty} \frac{6x-100}{8x+5} \stackrel{\ \text{ by LHR } \ }{=} \lim_{x\to\infty} \frac68 = \frac34.$$
    2. $$ \lim_{x\to \infty}\frac{e^x}{x^3} \stackrel{\ \text{ by LHR } \ }{=} \lim_{x\to\infty} \frac{e^x}{3x^2} \stackrel{\ \text{ by LHR } \ }{=} \lim_{x\to\infty} \frac{e^x}{6x} \stackrel{\ \text{ by LHR } \ }{=} \lim_{x\to\infty} \frac{e^x}{6} = \infty.$$ Recall that this means that the limit does not exist; as \(x\) approaches \(\infty\), the expression \(e^x/x^3\) grows without bound. We can infer from this that \(e^x\) grows "faster" than \(x^3\); as \(x\) gets large, \(e^x\) is far larger than \(x^3\). (This has important implications in computing when considering efficiency of algorithms.)

    L'Hôpital's Rule can only be applied to ratios of functions. When faced with an indeterminate form such as \(0\cdot\infty\) or \(\infty-\infty\), we can sometimes apply algebra to rewrite the limit so that L'Hôpital's Rule can be applied. We demonstrate the general idea in the next example.

    Example \(\PageIndex{3}\): Applying L'Hôpital's Rule to other indeterminate forms

    Evaluate the following limits.

    1. \( \lim_{x\to0^+} x\cdot e^{1/x}\)
    2. \( \lim_{x\to0^-} x\cdot e^{1/x}\)
    3. \( \lim_{x\to\infty} \ln(x+1)-\ln x\)
    4. \( \lim_{x\to\infty} x^2-e^x\)

    Solution

    1. As \(x\rightarrow 0^+\), \(x\rightarrow 0\) and \(e^{1/x}\rightarrow \infty\). Thus we have the indeterminate form \(0\cdot\infty\). We rewrite the expression \(x\cdot e^{1/x}\) as \(\frac{e^{1/x}}{1/x}\); now, as \(x\rightarrow 0^+\), we get the indeterminate form \(\infty/\infty\) to which L'Hôpital's Rule can be applied. $$ \lim_{x\to0^+} x\cdot e^{1/x} = \lim_{x\to 0^+} \frac{e^{1/x}}{1/x} \stackrel{\ \text{ by LHR } \ }{=} \lim_{x\to 0^+}\frac{(-1/x^2)e^{1/x}}{-1/x^2} =\lim_{x\to 0^+}e^{1/x} =\infty.$$ Interpretation: \(e^{1/x}\) grows "faster" than \(x\) shrinks to zero, meaning their product grows without bound.
    2. As \(x\rightarrow 0^-\), \(x\rightarrow 0\) and \(e^{1/x}\rightarrow e^{-\infty}\rightarrow 0\). The the limit evaluates to \(0\cdot 0\) which is not an indeterminate form. We conclude then that $$\lim_{x\to 0^-}x\cdot e^{1/x} = 0.\]
    3. This limit initially evaluates to the indeterminate form \(\infty-\infty\). By applying a logarithmic rule, we can rewrite the limit as
      $$ \lim_{x\to\infty} \ln(x+1)-\ln x = \lim_{x\to \infty} \ln \left(\frac{x+1}x\right).$$ As \(x\rightarrow \infty\), the argument of the \(\ln\) term approaches \(\infty/\infty\), to which we can apply L'Hôpital's Rule. $$\lim_{x\to\infty} \frac{x+1}x \stackrel{\ \text{ by LHR } \ }{=} \frac11=1.$$ Since \(x\rightarrow \infty\) implies \(\frac{x+1}x\rightarrow 1\), it follows that $$x\rightarrow \infty \quad \text{ implies }\quad \ln\left(\frac{x+1}x\right)\rightarrow \ln 1=0.\]

      Thus $$ \lim_{x\to\infty} \ln(x+1)-\ln x = \lim_{x\to \infty} \ln \left(\frac{x+1}x\right)=0.$$ Interpretation: since this limit evaluates to 0, it means that for large \(x\), there is essentially no difference between \(\ln (x+1)\) and \(\ln x\); their difference is essentially 0.

    4. The limit \( \lim_{x\to\infty} x^2-e^x\) initially returns the indeterminate form \(\infty-\infty\). We can rewrite the expression by factoring out \(x^2\); \( x^2-e^x = x^2\left(1-\frac{e^x}{x^2}\right).\) We need to evaluate how \(e^x/x^2\) behaves as \(x\rightarrow \infty\): $$\lim_{x\to\infty}\frac{e^x}{x^2} \stackrel{\ \text{ by LHR } \ }{=} \lim_{x\to\infty} \frac{e^x}{2x} \stackrel{\ \text{ by LHR } \ }{=} \lim_{x\to\infty} \frac{e^x}{2} = \infty.\]

      Thus \(\lim_{x\to\infty}x^2(1-e^x/x^2)\) evaluates to \(\infty\cdot(-\infty)\), which is not an indeterminate form; rather, \(\infty\cdot(-\infty)\) evaluates to \(-\infty\). We conclude that \( \lim_{x\to\infty} x^2-e^x = -\infty.\)

      Interpretation: as \(x\) gets large, the difference between \(x^2\) and \(e^x\) grows very large.

    Indeterminate Forms \(0^0\), \(1^\infty\) and \(\infty^0\)

    When faced with an indeterminate form that involves a power, it often helps to employ the natural logarithmic function. The following Key Idea expresses the concept, which is followed by an example that demonstrates its use.

    Key Idea 20: Evaluating Limits Involving Indeterminate Forms \(0^0\), \(1^\infty\) and \(\infty^0\)

    If \( \lim_{x\to c} \ln\big(f(x)\big) = L\), then \( \lim_{x\to c} f(x) = \lim_{x\to c} e^{\ln(f(x))} = e\,^L.\)

    Example \(\PageIndex{4}\): Using L'Hôpital's Rule with indeterminate forms involving exponents

    Evaluate the following limits.

    \( 1. \lim_{x\to\infty} \left(1+\frac1x\right)^x \qquad\qquad 2. \lim_{x\to0^+} x^x.\)

    Solution

    1. This equivalent to a special limit given in Theorem \ref{thm:lim_continuous}; these limits have important applications within mathematics and finance. Note that the exponent approaches \(\infty\) while the base approaches 1, leading to the indeterminate form \(1^\infty\). Let \(f(x) = (1+1/x)^x\); the problem asks to evaluate \(\lim_{x\to\infty}f(x)\). Let's first evaluate \( \lim_{x\to\infty}\ln\big(f(x)\big)\). \[\begin{align}\lim_{x\to\infty}\ln\big(f(x)\big) & = \lim_{x\to\infty} \ln \left(1+\frac1x\right)^x \\ &= \lim_{x\to\infty} x\ln\left(1+\frac1x\right)\\ &= \lim_{x\to\infty} \frac{\ln\left(1+\frac1x\right)}{1/x}\end{align}\] This produces the indeterminate form 0/0, so we apply L'Hôpital's Rule. \[\begin{align}&= \lim_{x\to\infty} \frac{\frac{1}{1+1/x}\cdot(-1/x^2)}{(-1/x^2)} \\&= \lim_{x\to\infty}\frac{1}{1+1/x}\\ &= 1. \end{align}\] Thus \(\lim_{x\to\infty} \ln \big(f(x)\big) = 1.\) We return to the original limit and apply Key Idea 20. $$\lim_{x\to\infty}\left(1+\frac1x\right)^x = \lim_{x\to\infty} f(x) = \lim_{x\to\infty}e^{\ln (f(x))} = e^1 = e.\]
    2. This limit leads to the indeterminate form \(0^0\). Let \(f(x) = x^x\) and consider first \(\lim_{x\to0^+} \ln\big(f(x)\big)\). \[\begin{align} \lim_{x\to0^+} \ln\big(f(x)\big) &= \lim_{x\to0^+} \ln\left(x^x\right) \\ &= \lim_{x\to0^+} x\ln x \\ &= \lim_{x\to0^+} \frac{\ln x}{1/x}.\end{align}\] This produces the indeterminate form \(-\infty/\infty\) so we apply L'Hôpital's Rule. \[\begin{align} &= \lim_{x\to0^+} \frac{1/x}{-1/x^2} \\ &= \lim_{x\to0^+} -x \\ &= 0. \end{align}\] Thus \(\lim_{x\to0^+} \ln\big(f(x)\big) =0\). We return to the original limit and apply Key Idea 20. $$\lim_{x\to0^+} x^x = \lim_{x\to0^+} f(x) = \lim_{x\to0^+} e^{\ln(f(x))} = e^0 = 1.$$ This result is supported by the graph of \(f(x)=x^x\) given in Figure \(\PageIndex{1}\).

    6.7: L'Hopital's Rule (2)

    Figure \(\PageIndex{1}\): A graph of \(f(x)=x^x\) supporting the fact that as \(x\to 0^+\), \(f(x)\to 1\).

    Our brief revisit of limits will be rewarded in the next section where we consider improper integration. So far, we have only considered definite integrals where the bounds are finite numbers, such as \(\int_0^1 f(x)\ dx\). Improper integration considers integrals where one, or both, of the bounds are "infinity." Such integrals have many uses and applications, in addition to generating ideas that are enlightening.

    Contributors and Attributions

    • Gregory Hartman (Virginia Military Institute).Contributions were made by Troy Siemers andDimplekumar Chalishajar of VMI and Brian Heinold of Mount Saint Mary's University. This content iscopyrighted by a Creative CommonsAttribution - Noncommercial (BY-NC) License.http://www.apexcalculus.com/

    • Integrated by Justin Marshall.

    6.7: L'Hopital's Rule (2024)

    FAQs

    What is 4.4 L Hospital's rule? ›

    CONCEPT: SECTION 4.4: L'HOSPITAL'S RULE

    Recall that this rule establishes that under certain circ*mstances, the limit of a ration of functions is equal to the limit of the ration of the derivatives of the functions. g′(x) , provide that the limit on the right-hand-side either exists or is ±∞.

    Can you use l'hôpital's rule multiple times? ›

    L Hospital rule can be applied more than once. You can apply this rule still it holds any indefinite form every time after its applications. If the problem is out of the indeterminate forms, you can't be able to apply L'Hospital's Rule.

    How do you calculate L Hopital's rule? ›

    What is L Hospital Rule Formula? The formula of L Hopitals rule is lim x a f(x) / g(x) = lim x a f' (x) / g'(x), where the left side limit gives an indeterminate form by applying the limit x = a.

    Can you use L'Hôpital's rule for infinity over zero? ›

    L'Hospital's Rule works great on the two indeterminate forms 0/0 and ±∞/±∞ ± ∞ / ± ∞ . However, there are many more indeterminate forms out there as we saw earlier. Let's take a look at some of those and see how we deal with those kinds of indeterminate forms.

    What is the stronger form of L Hopital's rule? ›

    L'Hô pital's Rule can be strengthened to include the case when g′(a)=0 and the indeterminate form " ∞/∞ ", the case when both f and g increase without any bound.

    Can you use L Hopital's rule for any indeterminate form? ›

    Indeterminate forms besides "0"0 and "∞"∞ include "0⋅∞, "∞−∞", "1∞", "00", and "∞0". These forms also arise in the computation of limits and can often be algebraically transformed into the form "0"0 or "∞"∞, so that l'Hopital's Rule can be applied.

    What limits does L Hopital's rule apply to? ›

    When Can You Use L'hopital's Rule. We can apply L'Hopital's rule, also commonly spelled L'Hospital's rule, whenever direct substitution of a limit yields an indeterminate form. This means that the limit of a quotient of functions (i.e., an algebraic fraction) is equal to the limit of their derivatives.

    How to solve infinity * 0 limit? ›

    0 is an indeterminate form, that is, the value can't be determined exactly. But, if we write it in the form of limits, then we see that: ⇒ limn n0 = limn 1 = 1. This is an example of the type limn f(x)g(x), and if it is known that f(x) = n or multiples of n and g(x) = 0, then it is a constant.

    Is 0 times infinity indeterminate? ›

    Zero Times Infinity

    The answer is undefined again! It could be any number that we can't predict. Many people make this mistake, they think that the answer is zero because anything multiplied by zero is zero but what they don't realize is the infinity sign with it.

    Who did L'Hôpital steal his rule from? ›

    The rule is named after the 17th-century French mathematician Guillaume De l'Hôpital. Although the rule is often attributed to De l'Hôpital, the theorem was first introduced to him in 1694 by the Swiss mathematician Johann Bernoulli.

    Can you use L Hopital's rule for sin? ›

    What is the limit limx→0sin(x)x? limx→0sin(x)x=1 . We determine this by the use of L'Hospital's Rule. Or in words, the limit of the quotient of two functions is equal to the limit of the quotient of their derivatives.

    Does L Hopital's rule apply to 1 0? ›

    l'Hopital's is true ONLY if you have a 0//0 or an ∞ / ∞ form. If you have any other form, it is not true.

    Can you use L Hopital's rule for asymptotes? ›

    Since L'Hôpital's Rule applies to limits where x → 소с, we can use it to find any horizontal asymptotes.

    Is it l'hospital or l'hôpital? ›

    However, with time going, many French spellings have been altered: in particular, the silent s has been removed and replaced with the circumflex over the preceding o making l'Hôpital. The former spelling is still used in English with no circumflex.

    What is the L hospital rule statement? ›

    Statement of L' Hospitals' Rule

    L' Hospital's rule states that, when the limit of f(x)g(x) is indeterminate, under certain conditions it can be obtained by evaluating the limit of the quotient of the derivatives of f and g (i.e., f′(x)g′(x)). If this result is indeterminate, the procedure can be repeated.

    When should you use L Hopital's rule? ›

    We can apply L'Hopital's rule, also commonly spelled L'Hospital's rule, whenever direct substitution of a limit yields an indeterminate form. This means that the limit of a quotient of functions (i.e., an algebraic fraction) is equal to the limit of their derivatives.

    What is the L hospital chain rule? ›

    If f(x) and g(x) are continuous functions with either f(a)=g(a)=0, or f(a)=±∞ and g(a)=±∞, then limx→af(x)g(x)=limx→af′(x)g′(x). Warning: While L'Hospital's Rule says that the limits of fg and f′g′ are equal under certain circ*mstances, it is not true that fg=f′g′!

    How do you know if L Hopital's rule applies? ›

    L'Hôpital's rule can only be applied in the case where direct substitution yields an indeterminate form, meaning 0/0 or ±∞/±∞. So if f and g are defined, L'Hôpital would be applicable only if the value of both f and g is 0.

    Top Articles
    Village Seven Presbyterian Church - The Gospel Coalition
    What we know about megachurch pastor Matt Chandler of The Village Church
    Jin Wigs Thomaston Ga
    scotty rasmussen paternity court
    Syracuse Pets Craigslist
    Kool Online Offender Lookup
    Greet In Cheshire Crossword Clue
    Bi State Schedule
    Pulse Point Oxnard
    Great Buildings Forge Of Empires
    Walmart Automotive Number
    Evil Dead Rise Showtimes Near Amc Antioch 8
    8776685260
    The Front Porch Self Service
    manhattan cars & trucks - by owner - craigslist
    Hydro Quebec Power Outage Map
    Trinket Of Advanced Weaponry
    Gay Pnp Zoom Meetings
    New & Used Motorcycles for Sale | NL Classifieds
    月曜から夜ふかし 9Tsu
    Leaks Mikayla Campinos
    Best Builder Hall 5 Base
    M3Gan Showtimes Near Regal City North
    Roses Gordon Highway
    Half Inning In Which The Home Team Bats Crossword
    Dmv Rocklin Wait Times
    Python Regex Space
    Jennifer Beals Bikini
    Stellaris Remove Planet Modifier
    Mapa i lokalizacja NPC w Graveyard Keeper - Graveyard Keeper - poradnik do gry | GRYOnline.pl
    Mega Millions Lottery - Winning Numbers & Results
    Best 43-inch TVs in 2024: Tested and rated
    2024-25 ITH Season Preview: USC Trojans
    Unveiling the World of Gimkit Hacks: A Deep Dive into Cheating
    Quattrocento, Italienische Kunst des 15. Jahrhunderts
    Hewn New Bedford
    Star Wars Galaxy Of Heroes Forums
    The QWERTY Keyboard Is Tech's Biggest Unsolved Mystery
    Ups Near Me Open
    Ella And David Steve Strange
    13 The Musical Common Sense Media
    Aid Office On 59Th Ashland
    When is the next full moon? September's Harvest Moon is also super
    Katie Hamden Of
    How Did Kratos Remove The Chains
    8569 Marshall St, Merrillville, IN 46410 - MLS 809825 - Coldwell Banker
    Where Is Katie Standon Now 2021
    Olive Onyx Amora
    Timothy Warren Cobb Obituary
    FINAL FANTASY XI Online 20th Anniversary | Square Enix Blog
    C Weather London
    Dominos Nijmegen Daalseweg
    Latest Posts
    Article information

    Author: Sen. Emmett Berge

    Last Updated:

    Views: 5745

    Rating: 5 / 5 (60 voted)

    Reviews: 83% of readers found this page helpful

    Author information

    Name: Sen. Emmett Berge

    Birthday: 1993-06-17

    Address: 787 Elvis Divide, Port Brice, OH 24507-6802

    Phone: +9779049645255

    Job: Senior Healthcare Specialist

    Hobby: Cycling, Model building, Kitesurfing, Origami, Lapidary, Dance, Basketball

    Introduction: My name is Sen. Emmett Berge, I am a funny, vast, charming, courageous, enthusiastic, jolly, famous person who loves writing and wants to share my knowledge and understanding with you.